0
$\begingroup$

I have the following nonlinear optimisation problem arising in my model.

$$\min \sum_{k=0}^{N-1} (\tau-t_k)^+\quad \text{ s.t. } {\mathbf{x}^\top\mathbf{w}\le W,\ \mathbf{x}\ge0}, t_k=t_{k-1}+x_k \text{ and } t_0=0.$$

I tried to simplify this problem by setting $\lambda_k=(\tau-t_k)^+$. I get the condition that $\rho w_i=\frac{N-i}{K_i}+\mu_i $, for all $i$, where $\rho$ and $\mu_i$ are appropriate Lagrange multipliers. How do I use this to find the optimal solution?

$\endgroup$
1
  • $\begingroup$ Are $t,x$ the problem variables? And I guess $(x)^+= \max(x,0)$, right? $\endgroup$ Commented Jan 1, 2016 at 14:35

1 Answer 1

2
$\begingroup$

Your problem is a linear problem: introducing auxiliary variables $z_k$ for $k=0,\ldots,,N-1$, you obtain

$$ \begin{align} &\min \sum_{k=0}^{N-1}z_k \\ &t_0 = 0\\ &t_{k} = t_{k-1} + x_k, \quad k=0,\ldots,N-1\\ &w^T x \leq W \\ &z_k\geq 0, \quad k=0,\ldots,N-1\\ &z_k\geq \tau - t_k, \quad k=0,\ldots,N-1\\ &x\geq 0 \end{align} $$

Any LP solver will do.

$\endgroup$

You must log in to answer this question.

Start asking to get answers

Find the answer to your question by asking.

Ask question

Explore related questions

See similar questions with these tags.